Stay ahead of learning milestones! Enroll in a class over the summer!

G
Topic
First Poster
Last Poster
k a May Highlights and 2025 AoPS Online Class Information
jlacosta   0
May 1, 2025
May is an exciting month! National MATHCOUNTS is the second week of May in Washington D.C. and our Founder, Richard Rusczyk will be presenting a seminar, Preparing Strong Math Students for College and Careers, on May 11th.

Are you interested in working towards MATHCOUNTS and don’t know where to start? We have you covered! If you have taken Prealgebra, then you are ready for MATHCOUNTS/AMC 8 Basics. Already aiming for State or National MATHCOUNTS and harder AMC 8 problems? Then our MATHCOUNTS/AMC 8 Advanced course is for you.

Summer camps are starting next month at the Virtual Campus in math and language arts that are 2 - to 4 - weeks in duration. Spaces are still available - don’t miss your chance to have an enriching summer experience. There are middle and high school competition math camps as well as Math Beasts camps that review key topics coupled with fun explorations covering areas such as graph theory (Math Beasts Camp 6), cryptography (Math Beasts Camp 7-8), and topology (Math Beasts Camp 8-9)!

Be sure to mark your calendars for the following upcoming events:
[list][*]May 9th, 4:30pm PT/7:30pm ET, Casework 2: Overwhelming Evidence — A Text Adventure, a game where participants will work together to navigate the map, solve puzzles, and win! All are welcome.
[*]May 19th, 4:30pm PT/7:30pm ET, What's Next After Beast Academy?, designed for students finishing Beast Academy and ready for Prealgebra 1.
[*]May 20th, 4:00pm PT/7:00pm ET, Mathcamp 2025 Qualifying Quiz Part 1 Math Jam, Problems 1 to 4, join the Canada/USA Mathcamp staff for this exciting Math Jam, where they discuss solutions to Problems 1 to 4 of the 2025 Mathcamp Qualifying Quiz!
[*]May 21st, 4:00pm PT/7:00pm ET, Mathcamp 2025 Qualifying Quiz Part 2 Math Jam, Problems 5 and 6, Canada/USA Mathcamp staff will discuss solutions to Problems 5 and 6 of the 2025 Mathcamp Qualifying Quiz![/list]
Our full course list for upcoming classes is below:
All classes run 7:30pm-8:45pm ET/4:30pm - 5:45pm PT unless otherwise noted.

Introductory: Grades 5-10

Prealgebra 1 Self-Paced

Prealgebra 1
Tuesday, May 13 - Aug 26
Thursday, May 29 - Sep 11
Sunday, Jun 15 - Oct 12
Monday, Jun 30 - Oct 20
Wednesday, Jul 16 - Oct 29

Prealgebra 2 Self-Paced

Prealgebra 2
Wednesday, May 7 - Aug 20
Monday, Jun 2 - Sep 22
Sunday, Jun 29 - Oct 26
Friday, Jul 25 - Nov 21

Introduction to Algebra A Self-Paced

Introduction to Algebra A
Sunday, May 11 - Sep 14 (1:00 - 2:30 pm ET/10:00 - 11:30 am PT)
Wednesday, May 14 - Aug 27
Friday, May 30 - Sep 26
Monday, Jun 2 - Sep 22
Sunday, Jun 15 - Oct 12
Thursday, Jun 26 - Oct 9
Tuesday, Jul 15 - Oct 28

Introduction to Counting & Probability Self-Paced

Introduction to Counting & Probability
Thursday, May 15 - Jul 31
Sunday, Jun 1 - Aug 24
Thursday, Jun 12 - Aug 28
Wednesday, Jul 9 - Sep 24
Sunday, Jul 27 - Oct 19

Introduction to Number Theory
Friday, May 9 - Aug 1
Wednesday, May 21 - Aug 6
Monday, Jun 9 - Aug 25
Sunday, Jun 15 - Sep 14
Tuesday, Jul 15 - Sep 30

Introduction to Algebra B Self-Paced

Introduction to Algebra B
Tuesday, May 6 - Aug 19
Wednesday, Jun 4 - Sep 17
Sunday, Jun 22 - Oct 19
Friday, Jul 18 - Nov 14

Introduction to Geometry
Sunday, May 11 - Nov 9
Tuesday, May 20 - Oct 28
Monday, Jun 16 - Dec 8
Friday, Jun 20 - Jan 9
Sunday, Jun 29 - Jan 11
Monday, Jul 14 - Jan 19

Paradoxes and Infinity
Mon, Tue, Wed, & Thurs, Jul 14 - Jul 16 (meets every day of the week!)

Intermediate: Grades 8-12

Intermediate Algebra
Sunday, Jun 1 - Nov 23
Tuesday, Jun 10 - Nov 18
Wednesday, Jun 25 - Dec 10
Sunday, Jul 13 - Jan 18
Thursday, Jul 24 - Jan 22

Intermediate Counting & Probability
Wednesday, May 21 - Sep 17
Sunday, Jun 22 - Nov 2

Intermediate Number Theory
Sunday, Jun 1 - Aug 24
Wednesday, Jun 18 - Sep 3

Precalculus
Friday, May 16 - Oct 24
Sunday, Jun 1 - Nov 9
Monday, Jun 30 - Dec 8

Advanced: Grades 9-12

Olympiad Geometry
Tuesday, Jun 10 - Aug 26

Calculus
Tuesday, May 27 - Nov 11
Wednesday, Jun 25 - Dec 17

Group Theory
Thursday, Jun 12 - Sep 11

Contest Preparation: Grades 6-12

MATHCOUNTS/AMC 8 Basics
Friday, May 23 - Aug 15
Monday, Jun 2 - Aug 18
Thursday, Jun 12 - Aug 28
Sunday, Jun 22 - Sep 21
Tues & Thurs, Jul 8 - Aug 14 (meets twice a week!)

MATHCOUNTS/AMC 8 Advanced
Sunday, May 11 - Aug 10
Tuesday, May 27 - Aug 12
Wednesday, Jun 11 - Aug 27
Sunday, Jun 22 - Sep 21
Tues & Thurs, Jul 8 - Aug 14 (meets twice a week!)

AMC 10 Problem Series
Friday, May 9 - Aug 1
Sunday, Jun 1 - Aug 24
Thursday, Jun 12 - Aug 28
Tuesday, Jun 17 - Sep 2
Sunday, Jun 22 - Sep 21 (1:00 - 2:30 pm ET/10:00 - 11:30 am PT)
Monday, Jun 23 - Sep 15
Tues & Thurs, Jul 8 - Aug 14 (meets twice a week!)

AMC 10 Final Fives
Sunday, May 11 - Jun 8
Tuesday, May 27 - Jun 17
Monday, Jun 30 - Jul 21

AMC 12 Problem Series
Tuesday, May 27 - Aug 12
Thursday, Jun 12 - Aug 28
Sunday, Jun 22 - Sep 21
Wednesday, Aug 6 - Oct 22

AMC 12 Final Fives
Sunday, May 18 - Jun 15

AIME Problem Series A
Thursday, May 22 - Jul 31

AIME Problem Series B
Sunday, Jun 22 - Sep 21

F=ma Problem Series
Wednesday, Jun 11 - Aug 27

WOOT Programs
Visit the pages linked for full schedule details for each of these programs!


MathWOOT Level 1
MathWOOT Level 2
ChemWOOT
CodeWOOT
PhysicsWOOT

Programming

Introduction to Programming with Python
Thursday, May 22 - Aug 7
Sunday, Jun 15 - Sep 14 (1:00 - 2:30 pm ET/10:00 - 11:30 am PT)
Tuesday, Jun 17 - Sep 2
Monday, Jun 30 - Sep 22

Intermediate Programming with Python
Sunday, Jun 1 - Aug 24
Monday, Jun 30 - Sep 22

USACO Bronze Problem Series
Tuesday, May 13 - Jul 29
Sunday, Jun 22 - Sep 1

Physics

Introduction to Physics
Wednesday, May 21 - Aug 6
Sunday, Jun 15 - Sep 14
Monday, Jun 23 - Sep 15

Physics 1: Mechanics
Thursday, May 22 - Oct 30
Monday, Jun 23 - Dec 15

Relativity
Mon, Tue, Wed & Thurs, Jun 23 - Jun 26 (meets every day of the week!)
0 replies
jlacosta
May 1, 2025
0 replies
Cheesy's math casino and probability
pithon_with_an_i   0
12 minutes ago
Source: Revenge JOM 2025 Problem 4, Revenge JOMSL 2025 C3
There are $p$ people are playing a game at Cheesy's math casino, where $p$ is a prime number. Let $n$ be a positive integer. A subset of length $s$ from the set of integers from $1$ to $n$ inclusive is randomly chosen, with an equal probability ($s \leq  n$ and is fixed). The winner of Cheesy's game is person $i$, if the sum of the chosen numbers are congruent to $i \pmod p$ for $0 \leq i \leq p-1$.
For each $n$, find all values of $s$ such that no person will sue Cheesy for creating unfair games (i.e. all the winning outcomes are equally likely).

(Proposed by Jaydon Chieng, Yeoh Teck En)

Remark
0 replies
pithon_with_an_i
12 minutes ago
0 replies
Partitioning coprime integers to arithmetic sequences
sevket12   4
N 13 minutes ago by bochidd
Source: 2025 Turkey EGMO TST P3
For a positive integer $n$, let $S_n$ be the set of positive integers that do not exceed $n$ and are coprime to $n$. Define $f(n)$ as the smallest positive integer that allows $S_n$ to be partitioned into $f(n)$ disjoint subsets, each forming an arithmetic progression.

Prove that there exist infinitely many pairs $(a, b)$ satisfying $a, b > 2025$, $a \mid b$, and $f(a) \nmid f(b)$.
4 replies
sevket12
Feb 8, 2025
bochidd
13 minutes ago
Coaxal Circles
fattypiggy123   30
N 15 minutes ago by Ilikeminecraft
Source: China TSTST Test 2 Day 1 Q3
Let $ABCD$ be a quadrilateral and let $l$ be a line. Let $l$ intersect the lines $AB,CD,BC,DA,AC,BD$ at points $X,X',Y,Y',Z,Z'$ respectively. Given that these six points on $l$ are in the order $X,Y,Z,X',Y',Z'$, show that the circles with diameter $XX',YY',ZZ'$ are coaxal.
30 replies
fattypiggy123
Mar 13, 2017
Ilikeminecraft
15 minutes ago
Weird n-variable extremum problem
pithon_with_an_i   0
19 minutes ago
Source: Revenge JOM 2025 Problem 3, Revenge JOMSL 2025 A4
Let $n$ be a positive integer greater or equal to $2$ and let $a_1$, $a_2$, ..., $a_n$ be a sequence of non-negative real numbers. Find the maximum value of $3(a_1  + a_2 + \cdots + a_n) - (a_1^2 + a_2^2 + \cdots + a_n^2) - a_1a_2  \cdots a_n$ in terms of $n$.

(Proposed by Cheng You Seng)
0 replies
pithon_with_an_i
19 minutes ago
0 replies
Inequalities
sqing   3
N 5 hours ago by sqing
Let $a,b,c >2 $ and $ ab+bc+ca \leq 75.$ Show that
$$\frac{1}{a-2}+\frac{1}{b-2}+\frac{1}{c-2}\geq 1$$Let $a,b,c >2 $ and $ \frac{1}{a}+\frac{1}{b}+\frac{1}{c}\geq \frac{6}{7}.$ Show that
$$\frac{1}{a-2}+\frac{1}{b-2}+\frac{1}{c-2}\geq 2$$
3 replies
sqing
Yesterday at 11:31 AM
sqing
5 hours ago
Assam Mathematics Olympiad 2022 Category III Q18
SomeonecoolLovesMaths   2
N 6 hours ago by nyacide
Let $f : \mathbb{N} \longrightarrow \mathbb{N}$ be a function such that
(a) $ f(m) < f(n)$ whenever $m < n$.
(b) $f(2n) = f(n) + n$ for all $n \in \mathbb{N}$.
(c) $n$ is prime whenever $f(n)$ is prime.
Find $$\sum_{n=1}^{2022} f(n).$$
2 replies
SomeonecoolLovesMaths
Sep 12, 2024
nyacide
6 hours ago
Assam Mathematics Olympiad 2022 Category III Q17
SomeonecoolLovesMaths   1
N Today at 7:24 AM by nyacide
Consider a rectangular grid of points consisting of $4$ rows and $84$ columns. Each point is coloured with one of the colours red, blue or green. Show that no matter whatever way the colouring is done, there always exist four points
of the same colour that form the vertices of a rectangle. An illustration is shown in the figure below.
1 reply
SomeonecoolLovesMaths
Sep 12, 2024
nyacide
Today at 7:24 AM
Assam Mathematics Olympiad 2022 Category III Q14
SomeonecoolLovesMaths   1
N Today at 6:54 AM by nyacide
The following sum of three four digits numbers is divisible by $75$, $7a71 + 73b7 + c232$, where $a, b, c$ are decimal digits. Find the necessary conditions in $a, b, c$.
1 reply
SomeonecoolLovesMaths
Sep 12, 2024
nyacide
Today at 6:54 AM
Assam Mathematics Olympiad 2022 Category III Q12
SomeonecoolLovesMaths   2
N Today at 6:20 AM by nyacide
A particle is in the origin of the Cartesian plane. In each step the particle can go $1$ unit in any of the directions, left, right, up or down. Find the number of ways to go from $(0, 0)$ to $(0, 2)$ in $6$ steps. (Note: Two paths where identical set of points is traversed are considered different if the order of traversal of each point is different in both paths.)
2 replies
SomeonecoolLovesMaths
Sep 12, 2024
nyacide
Today at 6:20 AM
Assam Mathematics Olympiad 2022 Category III Q10
SomeonecoolLovesMaths   1
N Today at 5:53 AM by nyacide
Let the vertices of the square $ABCD$ are on a circle of radius $r$ and with center $O$. Let $P, Q, R$ and $S$ are the mid points of $AB, BC, CD$ and $DA$ respectively. Then;
(a) Show that the quadrilateral $P QRS$ is a square.
(b) Find the distance from the mid point of $P Q$ to $O$.
1 reply
SomeonecoolLovesMaths
Sep 12, 2024
nyacide
Today at 5:53 AM
A problem of collinearity.
Raul_S_Baz   2
N Today at 4:11 AM by Raul_S_Baz
Î am the author.
IMAGE
P.S: How can I verify that it is an original problem? Thanks!
2 replies
Raul_S_Baz
Yesterday at 4:19 PM
Raul_S_Baz
Today at 4:11 AM
Inequalities
sqing   0
Today at 3:46 AM
Let $ a,b,c>0 $ . Prove that
$$\frac{a+5b}{b+c}+\frac{b+5c}{c+a}+\frac{c+5a}{a+b}\geq 9$$$$ \frac{2a+11b}{b+c}+\frac{2b+11c}{c+a}+\frac{2c+11a}{a+b}\geq \frac{39}{2}$$$$ \frac{25a+147b}{b+c}+\frac{25b+147c}{c+a}+\frac{25c+147a}{a+b} \geq258$$
0 replies
sqing
Today at 3:46 AM
0 replies
Plz help
Bet667   3
N Yesterday at 6:50 PM by K1mchi_
f:R-->R for any integer x,y
f(yf(x)+f(xy))=(x+f(x))f(y)
find all function f
(im not good at english)
3 replies
Bet667
Jan 28, 2024
K1mchi_
Yesterday at 6:50 PM
2019 SMT Team Round - Stanford Math Tournament
parmenides51   17
N Yesterday at 6:40 PM by Rombo
p1. Given $x + y = 7$, find the value of x that minimizes $4x^2 + 12xy + 9y^2$.


p2. There are real numbers $b$ and $c$ such that the only $x$-intercept of $8y = x^2 + bx + c$ equals its $y$-intercept. Compute $b + c$.



p3. Consider the set of $5$ digit numbers $ABCDE$ (with $A \ne 0$) such that $A+B = C$, $B+C = D$, and $C + D = E$. What’s the size of this set?


p4. Let $D$ be the midpoint of $BC$ in $\vartriangle ABC$. A line perpendicular to D intersects $AB$ at $E$. If the area of $\vartriangle ABC$ is four times that of the area of $\vartriangle BDE$, what is $\angle ACB$ in degrees?


p5. Define the sequence $c_0, c_1, ...$ with $c_0 = 2$ and $c_k = 8c_{k-1} + 5$ for $k > 0$. Find $\lim_{k \to \infty} \frac{c_k}{8^k}$.


p6. Find the maximum possible value of $|\sqrt{n^2 + 4n + 5} - \sqrt{n^2 + 2n + 5}|$.


p7. Let $f(x) = \sin^8 (x) + \cos^8(x) + \frac38 \sin^4 (2x)$. Let $f^{(n)}$ (x) be the $n$th derivative of $f$. What is the largest integer $a$ such that $2^a$ divides $f^{(2020)}(15^o)$?


p8. Let $R^n$ be the set of vectors $(x_1, x_2, ..., x_n)$ where $x_1, x_2,..., x_n$ are all real numbers. Let $||(x_1, . . . , x_n)||$ denote $\sqrt{x^2_1 +... + x^2_n}$. Let $S$ be the set in $R^9$ given by $$S = \{(x, y, z) : x, y, z \in R^3 , 1 = ||x|| = ||y - x|| = ||z -y||\}.$$If a point $(x, y, z)$ is uniformly at random from $S$, what is $E[||z||^2]$?


p9. Let $f(x)$ be the unique integer between $0$ and $x - 1$, inclusive, that is equivalent modulo $x$ to $\left( \sum^2_{i=0} {{x-1} \choose i} ((x - 1 - i)! + i!) \right)$. Let $S$ be the set of primes between $3$ and $30$, inclusive. Find $\sum_{x\in S}^{f(x)}$.


p10. In the Cartesian plane, consider a box with vertices $(0, 0)$,$\left( \frac{22}{7}, 0\right)$,$(0, 24)$,$\left( \frac{22}{7}, 4\right)$. We pick an integer $a$ between $1$ and $24$, inclusive, uniformly at random. We shoot a puck from $(0, 0)$ in the direction of $\left( \frac{22}{7}, a\right)$ and the puck bounces perfectly around the box (angle in equals angle out, no friction) until it hits one of the four vertices of the box. What is the expected number of times it will hit an edge or vertex of the box, including both when it starts at $(0, 0)$ and when it ends at some vertex of the box?


p11. Sarah is buying school supplies and she has $\$2019$. She can only buy full packs of each of the following items. A pack of pens is $\$4$, a pack of pencils is $\$3$, and any type of notebook or stapler is $\$1$. Sarah buys at least $1$ pack of pencils. She will either buy $1$ stapler or no stapler. She will buy at most $3$ college-ruled notebooks and at most $2$ graph paper notebooks. How many ways can she buy school supplies?


p12. Let $O$ be the center of the circumcircle of right triangle $ABC$ with $\angle ACB = 90^o$. Let $M$ be the midpoint of minor arc $AC$ and let $N$ be a point on line $BC$ such that $MN \perp BC$. Let $P$ be the intersection of line $AN$ and the Circle $O$ and let $Q$ be the intersection of line $BP$ and $MN$. If $QN = 2$ and $BN = 8$, compute the radius of the Circle $O$.


p13. Reduce the following expression to a simplified rational $$\frac{1}{1 - \cos \frac{\pi}{9}}+\frac{1}{1 - \cos \frac{5 \pi}{9}}+\frac{1}{1 - \cos \frac{7 \pi}{9}}$$

p14. Compute the following integral $\int_0^{\infty} \log (1 + e^{-t})dt$.


p15. Define $f(n)$ to be the maximum possible least-common-multiple of any sequence of positive integers which sum to $n$. Find the sum of all possible odd $f(n)$


PS. You should use hide for answers. Collected here.
17 replies
parmenides51
Feb 6, 2022
Rombo
Yesterday at 6:40 PM
Russian NT with a Ceiling
naman12   45
N Apr 24, 2025 by InterLoop
Source: 2019 ISL N8
Let $a$ and $b$ be two positive integers. Prove that the integer
\[a^2+\left\lceil\frac{4a^2}b\right\rceil\]is not a square. (Here $\lceil z\rceil$ denotes the least integer greater than or equal to $z$.)

Russia
45 replies
naman12
Sep 22, 2020
InterLoop
Apr 24, 2025
Russian NT with a Ceiling
G H J
Source: 2019 ISL N8
The post below has been deleted. Click to close.
This post has been deleted. Click here to see post.
TechnoLenzer
55 posts
#34 • 1 Y
Y by pavel kozlov
Assume $(a+n)^2 = a^2 + \left\lceil\frac{4a^2}b\right\rceil$.

Part 1: Conversion into algebra

\begin{align*}
&\iff 2an + n^2 = \left\lceil\frac{4a^2}b\right\rceil \\
&\iff 2an + n^2 \geq \frac{4a^2}b> 2an + n^2 - 1 \\
&\iff \frac{4a^2}{2an + n^2 - 1} > b \ge \frac{4a^2}{2an + n^2}. \tag{1} \\
\end{align*}
Claim 2: $\frac{4a^2}{2an + n^2} = \frac{2a}{n} - \frac{2a}{2a + n}$.

Proof:
\begin{align*}
\frac{2a}{n} - \frac{2a}{2a + n} &= \frac{4a^2 + 2an}{n(2a+n)} - \frac{2an}{n(2a+n)} \\
&= \frac{4a^2 + 2an - 2an}{n(2a + n)} = \frac{4a^2}{2an + n^2}. \quad \square \\
\end{align*}
Claim 3: $\frac{4a^2}{2an + n^2 - 1} = \frac{2a}{n} - \frac{2a}{n} \left( \frac{n^2 - 1}{2an + n^2 - 1} \right)$.

Proof:
\begin{align*}
\frac{2a}{n} \left (1 - \frac{n^2 - 1}{2an + n^2 - 1} \right) &= \frac{2a}{n} \left( \frac{2an + (n^2 - 1) - (n^2 - 1)}{2an + n^2 - 1} \right) \\
&= \frac{2a}{n} \left( \frac{2an}{2an + n^2 - 1} \right) = \frac{4a^2}{2an + n^2 - 1}. \quad \square \\
\end{align*}
Claim 4: $n \le 2a$.

Proof: $\left\lceil\frac{4a^2}b\right\rceil \le 4a^2$ for $b \in \mathbb{N}$. Hence, $(a+n)^2 = a^2 + \left\lceil\frac{4a^2}b\right\rceil \le 5a^2 \le 9a^2 \Rightarrow a + n \le 3a \Rightarrow n \le 2a$. $\square$

Let $d = 2a$. By $(1)$ and Claims 2, 3,
\begin{align*}
\frac{d}{n} - \frac{d}{n}\left( \frac{n^2 - 1}{dn + n^2 - 1} \right) > b \ge \frac{d}{n} - \frac{d}{d+n} \tag{5}
\end{align*}
By Claim 4, we can let $d = kn - c$, for $k \in \mathbb{N} \ge 2$, $0 \le c < n$.

Now $(5)$ becomes:
\begin{align*}
k - \frac{c}{n} - \frac{kn - c}{n} \left( \frac{n^2 - 1}{kn^2 - cn + n^2 - 1} \right) &> b \ge k - \frac{c}{n} - \frac{kn-c}{(k+1)n - c} \\
\Rightarrow - \frac{c}{n} - \frac{kn - c}{n} \left( \frac{n^2 - 1}{kn^2 - cn + n^2 - 1} \right) &> b - k \ge - \frac{c}{n} - \frac{kn-c}{(k+1)n - c} \tag{6}
\end{align*}
$0 \le c < n \Rightarrow 0 \le \frac{c}{n} < 1$, and furthermore $kn-c < (k+1)n-c$ so $0 \le \frac{kn-c}{(k+1)n-c} < 1$. Thus, $-2 < RHS \le 0$.

Similarly, $n^2 \ge 1 \Rightarrow \frac{n^2 - 1}{kn^2 - cn + n^2 - 1} \ge 0$. Thus, $LHS \le 0$, but $LHS > RHS$ so $LHS > -2$ as well.

Part 2: Fixing bounds between integers

Now, it suffices to show that
\begin{align*}
\frac{kn-c}{n} \left( \frac{n^2 - 1}{(k+1)n^2 - cn - 1} \right) - \frac{n-c}{n}&\ge 0 \tag{7} \\
\iff \frac{kn-c}{(k+1)n - c} - \frac{n-c}n &\ge 0 \tag{8}
\end{align*}and that neither expression equals zero, for $n \in \mathbb{Z}$, $0 \le c < n$, because this implies

\begin{align*}
- \frac{c}{n} - \frac{kn - c}{n} \left( \frac{n^2 - 1}{(k+1)n^2 -cn - 1} \right) &\le -1 \\
\iff - \frac{kn - c}{n} \left( \frac{n^2 - 1}{(k+1)n^2 -cn - 1} \right) &\le - \frac{n-c}{n} \\
\iff \frac{kn - c}{n} \left( \frac{n^2 - 1}{(k+1)n^2 -cn - 1} \right) &\ge \frac{n-c}{n} \\
\iff \frac{kn-c}{(k+1)n - c} - \frac{n-c}n &\ge 0 \\
\iff - \frac{kn-c}{(k+1)n - c} &\le - \frac{n-c}n \\
\iff - \frac{c}{n} - \frac{kn-c}{(k+1)n - c} &\le -1
\end{align*}so both bounds always lie within either $(-2, -1)$ or $[-1, 0]$, and hence $b-k$ does too. However, note that $0 \ge LHS > b-k$, so $b-k \neq 0$. Furthermore, if either bound is -1, (7) or (8) must be equality, which contradicts our claim. Thus, $b-k$ ends up in $(-2, -1)$ or $(-1, 0)$, so cannot be an integer. But $b-k \in \mathbb{Z} \iff b \in \mathbb{Z}$ since $k$ integer, so $b \not \in \mathbb{Z}$, which is a contradiction.

Part 3: Proof of equivalence

\begin{align*}
(7) \iff (kn-c)(n^2-1) - (n-c)((k+1)n^2 - cn - 1) &\ge 0 \\
\iff kn^3 - cn^2 - kn + c - (k+1)n^3 + c(k+1)n^2 + cn^2 - c^2n + n - c &\ge 0 \\
\iff -kn - n^3 + c(k+1)n^2 - c^2n + n &\ge 0 \\
\iff -n^2 + c(k+1)n + (1 - k - c^2) &\ge 0. \tag{9}
\end{align*}
\begin{align*}
(8) \iff n(kn-c) - ((k+1)n - c)(n-c) &\ge 0 \\
\iff kn^2 - cn - (k+1)n^2 + c(k+1)n + cn - c^2 &\ge 0 \\
\iff -n^2 + c(k+1)n - c^2 &\ge 0. \tag{10}
\end{align*}
\begin{align*}
(9) \iff n &\in \left [ \frac{-c(k+1) + \sqrt{c^2(k+1)^2 + 4(c^2 + k - 1)}}{-2}, \frac{-c(k+1) - \sqrt{c^2(k+1)^2 + 4(c^2 + k - 1)}}{-2} \right] \\
\iff n &\in \left [ \frac{c}{2} \left((k+1) - \sqrt{(k+1)^2 + 4\left (1 + \frac{k-1}{c^2} \right)} \right), \frac{c}{2} \left((k+1) + \sqrt{(k+1)^2 + 4(1 + \frac{k-1}{c^2})} \right) \right].
\end{align*}Note that the lower bound $< \frac{c}{2} ((k+1) - \sqrt{(k+1)^2}) = 0$, so

$(9) \iff n \le \frac{c}{2} \left((k+1) + \sqrt{(k+1)^2 + 4(1 + \frac{k-1}{c^2})} \right)$.

Similarly, $(10) \iff n \le \frac{c}{2} ((k+1) + \sqrt{(k+1)^2 + 4})$, but because $k \ge 2$, all differences of two squares are $\ge 7$, and hence $(k+1)^2 + 4 < (k+2)^2$. Thus, $(10) \iff n \le \frac{c}{2} ((k+1) + (k+1)) = c(k+1)$.

Now
\begin{align*}
n &> c(k+1) \\
\Rightarrow (k+1)^2 + 4\left (1 + \frac{k-1}{c^2} \right) &\ge (k+3)^2 \\
\Rightarrow 4 + 4 \cdot \frac{k-1}{c^2} &\ge 4k + 8 \\
\Rightarrow \frac{k-1}{c^2} &\ge k + 1 \tag{11}
\end{align*}If $c = 0$, (9) and (10) both yield $-n^2 \ge 0$, a contradiction as $n \in \mathbb{Z}$.

If $c \ge 1$, (11) is false.

Part 4: Proof that equality is impossible

For (10), as above, $(k+1)^2 < (k+1)^2 + 4 < (k+2)^2$ and so $\sqrt{(k+1)^2 + 4}$ isn't an integer, and $n$ isn't either. Contradiction.

For (11), if $c = 0$, $n = 0$, contradiction. If $c = 1$, $(k+1)^2 < (k+1)^2 + 4k < (k+3)^2$, so $(k+1)^2 + 4k = (k+2)^2 \Rightarrow 2k = 3$, also contradiction. If $c \ge 2$, $(k+1)^2 < (k+1)^2 + 4\left (1 + \frac{k-1}{c^2} \right) < (k+1)^2 + k < (k+2)^2$, so once again $n$ isn't an integer.
This post has been edited 1 time. Last edited by TechnoLenzer, May 10, 2022, 7:00 PM
Z K Y
The post below has been deleted. Click to close.
This post has been deleted. Click here to see post.
ZETA_in_olympiad
2211 posts
#35
Y by
Assume the contrary then $a^2+\tfrac{4a^2+k}{b}=n^2$ with $b>k.$ Set $n=\tfrac{x+y}{2}$ and $a=\tfrac{x-y}{2}$ thus $x^2+y^2-$ $bxy-2xy+k=0.$ Fix $b$ and $k$ and take positive integer solutions $(x,y)$ such that $x+y$ is minimal. WLOG $x\geq y.$ By vieta jumping we have another solution $(z,y)=$ $(\tfrac{y^2+k}{x},y)=$ $((b+2)y-x,y).$ Note that $z$ is also a positive integer. Note that $x^2-y^2>(x-y)^2.$ But we have $b>k=bxy-(x-y)^2$ which implies $(x-y)^2>b>k.$ So $x>\tfrac{y^2+k}{x}= z,$ a contradiction with minimality.
This post has been edited 2 times. Last edited by ZETA_in_olympiad, Jun 26, 2022, 9:34 AM
Z K Y
The post below has been deleted. Click to close.
This post has been deleted. Click here to see post.
JAnatolGT_00
559 posts
#36
Y by
Assume the opposite, so $a^2+\frac{4a^2+\epsilon}{b}=c^2$ for $c\in \mathbb{Z}^+,\epsilon \in [0;b)$. Let $m=c-a,n=c+a$, so $$(n-m)^2+\epsilon =bmn\implies n^2-(2+b)mn+(m^2+\epsilon )=0 \text{ } (\bigstar)$$Note that $n_0=(2+b)-n=\frac{m^2+\epsilon}{n}\in \mathbb{Z}^+,$ so minimizing equality $\bigstar$ by $n$ we obtain $$n\leq \frac{m^2+\epsilon}{n}\implies 4a^2<4ac=n^2-m^2\leq \epsilon<b.$$Therefore $c^2=a^2+1,$ which is absurd, thus the contradiction.
Z K Y
The post below has been deleted. Click to close.
This post has been deleted. Click here to see post.
akasht
84 posts
#37 • 1 Y
Y by sabkx
We shall show the related expression $a^2+4\left\lceil\frac{a^2}b\right\rceil$ can never be a perfect square. Suppose otherwise, and consider the smallest $a$ for which such an expression is a perfect square, and say $a^2+4\left\lceil\frac{a^2}b\right\rceil=(a+y)^2$. Then $4\left\lceil\frac{a^2}b\right\rceil = 2ay + y^2$. Clearly $y$ is even, so letting $y=2x$ we get that $\left \lceil \frac{a^2}{b} \right \rceil = ax+x^2$

Claim: $b= \left \lfloor \frac{a}{x} \right \rfloor$

Proof: We have the inequalities $b(ax+x^2-1) < 4a^2 \le b(ax+x^2)$ Observe that \[\frac{a}{x} \cdot (ax+x^2-1) = a^2+ax - \frac{a}{x} \ge a^2\]implying $b \le \frac{a}{x}$. On the other hand, \[(\frac{a}{x} -1)(ax+x^2) = a^2 - x^2 < a^2 \implies b > \frac{a}{x}-1\]Since $b$ is an integer, $b= \left \lfloor \frac{a}{x} \right \rfloor$ as desired.

Next, let $a=bx+r$. Then \[\left\lceil\frac{a^2}b\right\rceil  = ax+x^2 \implies \left\lceil\frac{b^2x^2+2bxr+r^2}{b} \right\rceil  = (bx+r)x+x^2 \implies bx^2+2rx+ \left \lceil \frac{r^2}{b} \right \rceil = bx^2 + rx + x^2 \implies x^2-rx-\left \lceil \frac{r^2}{b} \right \rceil=0 \]
For the equation to have integer solutions in $x$, the discriminant of this quadratic must be a perfect square, in other words $r^2+4\left\lceil \frac{r^2}{b} \right \rceil$ is a perfect square. But $r<a$, contradicting the minimality of $a$! Thus $a^2+4\left\lceil\frac{a^2}b\right\rceil$ is never a perfect square.

To finish, observe that if $a^2+\left \lceil \frac{4a^2}{b} \right \rceil$ is a perfect square then $4a^2 + 4 \left \lceil \frac{4a^2}{b} \right \rceil = (2a)^2 + 4 \left \lceil \frac{(2a)^2}{b} \right \rceil$ is a perfect square, contradicting the earlier result.
This post has been edited 2 times. Last edited by akasht, Nov 3, 2022, 7:01 AM
Reason: Fixed LaTeX
Z K Y
The post below has been deleted. Click to close.
This post has been deleted. Click here to see post.
Olympikus
87 posts
#38
Y by
Suppose it equals $(a+k)^2$ ($k$ must be positive integer). First, I claim that $b = \left \lfloor \dfrac{2a}{k}\right \rfloor$. Note that the problem condition is equivalent to
\[-1< \frac{4a^2}{b}-k^2-2ak \leqslant 0\]\[\iff \frac{4a^2}{k^2+2ak} \leqslant b < \frac{4a^2}{k^2+2ak-1} \ \ \ \ \ \  (\clubsuit)\]But $\dfrac{4a^2}{k^2+2ak-1} \le \dfrac{2a}{k}$, so $b\leqslant \lfloor \dfrac{2a}{k}\rfloor$.
Now it suffices to prove that $b>\dfrac{2a}{k}-1$. Using the left inequality of $(\clubsuit)$,

\[b-\frac{2a}{k}\ge\frac{4a^2}{k^2+2ak} - \frac{4a^2}{2ak} = -\frac{4a^2k^2}{2ak(k^2+2ak)}>-1.\]This concludes the proof of the claim. Now write the euclidean division of $2a$ by $k$:
\[2a = kb+r,\ 0\le r<k\]\[\implies k^2+2ak = \left\lceil \frac{(kb+r)^2}{b} \right \rceil =\left\lceil \frac{r^2}{b} \right \rceil+k^2b+2kr \]\[\implies k^2+(kb+r)k = \left\lceil \frac{r^2}{b} \right \rceil+k^2b+2kr \]\[\implies k^2-kr =\left\lceil \frac{r^2}{b} \right \rceil\]So in particular $r$ is positive. Now looking at the discriminant (as a quadratic on $k$), we get that
\[r^2+4\left\lceil \frac{r^2}{b} \right \rceil = (r+2l)^2.\]Let $x>0$ be minimum such that $x^2+4\left\lceil \frac{x^2}{b} \right \rceil$ is a square.

\[\implies x^2+4\left\lceil \frac{x^2}{b} \right \rceil = (x+2y)^2\]
with $y>0$. Hence,

\[\left\lceil \frac{x^2}{b} \right \rceil = xy+y^2\]I claim $b = \lfloor\frac{x}{y}\rfloor$.
\[xy+y^2-1 < \frac{x^2}{b} \leqslant xy+y^2\]\[\frac{x^2}{xy+y^2}\leqslant b < \frac{x^2}{xy+y^2-1}\]\[\frac{x}{y}-1 <\frac{x^2}{xy+y^2}\leqslant b < \frac{x^2}{xy+y^2-1}\leqslant \frac{x}{y} \ \ \ \square\]
So we can divide $x$ by $y$:
\[x = by+c.\]
\[\left\lceil \frac{(by+c)^2}{b} \right \rceil = (by+c)y+y^2\]\[\left\lceil \frac{c^2}{b} \right \rceil = y^2-cy\]by the quadratic formula,
\[c^2-4\left\lceil \frac{c^2}{b} \right \rceil\]is a square, a contradiction if $x>c>0$. But $b,y>0 \implies c<x$ so $c=0$.

\[\implies xy = \left\lceil \frac{x^2}{b} \right \rceil = xy+y^2\]
a contradiction.
Z K Y
The post below has been deleted. Click to close.
This post has been deleted. Click here to see post.
IAmTheHazard
5001 posts
#39
Y by
I groupsolved this many years ago but here I am again.

Suppose that it does equal a square, i.e.
$$a^2+\left\lceil \frac{4a^2}{b}\right\rceil=c^2 \implies \left\lceil \frac{4a^2}{b}\right\rceil=c^2-a^2 \implies b(c^2-a^2-1)<4a^2\leq b(c^2-a^2).$$Motivated by the many squares, we perform the classic substitution $c+a= x$ and $c-a= y$, so $a=\tfrac{x-y}{2}$ and the inequality miraculously becomes
$$bxy-b<(x-y)^2\leq bxy.$$Thus $(x-y)^2=bxy-d \implies x^2-(b+2)xy+y^2+d=0$ where $0 \leq d < b$.
We now apply the technique of Vieta jumping. For fixed $b$ and $d$, suppose that there exist solutions in $\mathbb{Z}^+$ to the equation. Consider the pair $(x_1,y)$ of positive integers such that $x_1+y$ is minimal, and WLOG let $x_1>y$ ($x=y$ means $0>bxy-b\geq 0$: absurd). Viewing the equation as a quadratic in $x$ with $y$ fixed, we have two roots: one of which is $x_1$ and the other $x_2$. Since $x_1+x_2$ and $x_1x_2$ are both positive integers, $x_2$ is a positive integer as well, so by minimality $x_2\geq x_1>y$. Now we have
$$x_1+x_2=(b+2)y \text{ and } x_1x_2=y^2+d<y^2+b.$$Since $x_1x_2$ increases if $x_1+x_2$ are fixed and $x_1$ and $x_2$ approach each other, from the first equation and the fact that $x_1\geq y$,
$$x_1x_2\geq (b+1)y^2,$$so $y^2+b>(b+1)y^2 \implies b>by^2$, which is absurd. Hence no solutions can exist, implying that the expression given in the problem statement can never be a perfect square. $\blacksquare$
Z K Y
The post below has been deleted. Click to close.
This post has been deleted. Click here to see post.
Leo.Euler
577 posts
#40
Y by
Suppose that \[a^2+\left\lceil\frac{4a^2}b\right\rceil\]is a perfect square for some choice of $a$ and $b$. Then, this is \[ a^2 + \frac{4a^2+m}{b} = k^2 \]for some $0 \le m < b$. This can be rewritten as \[ (b+4)a^2 - bk^2 = -m. \]Using the well-known substitution $k = \frac{x+y}{2}$ and $a = \frac{x-y}{2}$, this becomes a Diophantine equation with the LHS a quadratic form in $x$ and $y$: \[ x^2 - (b+2)xy + y^2 = -m. \]It suffices to prove that this has no solutions for $x \ge y \ge 1$, which can be shown using Vieta jumping. Define the transform $T$ so that \[ T(x,y) =\left( y, \frac{y^2+m}{x} \right) = (y, (b+2)y-x)\]for $x, y \ge 1$; note that $T(x, y)$ is a solution to $x^2 - (b+2)xy + y^2 = -m$ if $(x, y)$ is a solution. If $xy \ge 2$, then \[ (x-y)^2 > bxy-m > bxy-b \ge b, \]from which \[ x^2-y^2 = (x-y)(x+y) > (x-y)^2 > b > m. \]Thus, $0 < \frac{y^2+m}{x} < x$ for all $(x, y)$ with $xy \ge 2$.

Now, for a given solution $(x, y)$ with $xy \ge 2$, repeatedly apply $T$; each time, a valid solution $(x_n, y_n)$ with $x_n, y_n \ge 1$ and $\max(x_n, y_n)<\max(x_{n-1}, y_{n-1})$ is generated. Clearly an infinitude of solutions is obtained by this. However, the maximum of the numbers in the pair eventually hits $1$, after which no solution can be generated. Thus no $(x, y)$ with $xy \ge 2$ works. Hence, $(x, y)=(1, 1)$ is the only possible solution, but this implies $b=m$, so it doesn't work. All in all, there are no solutions $(a, b)$ to the original Diophantine equation, as desired.
This post has been edited 4 times. Last edited by Leo.Euler, Apr 14, 2023, 1:40 AM
Z K Y
The post below has been deleted. Click to close.
This post has been deleted. Click here to see post.
megarnie
5608 posts
#41
Y by
Suppose it was a perfect square for some $a,b$.

Let $\frac{4a^2 + c}{b} = \left \lceil \frac{4a^2}{b} \right\rceil $, where $0\le c< b$.

The equation becomes \[a^2 + \frac{4a^2 + c}{b} =  d^2,\]where $d$ is a positive integer.

Multiplying both sides by $b$ gives $(b+4)a^2  - b d^2 = -c$.

Obviously $d>a$, so let $x = d+a$, and $y = d-a$, so that $d = \frac{x+y}{2}$ and $a = \frac{x-y}{2}$ and $x,y$ are positive integers.

After multiplying both sides by $4$, the equation becomes $(b+4)(x-y)^2 - b (x+y)^2  = -4c$, so \[ x^2 - (b+2)xy + y^2 = -c\]
Notice that if $(x,y)$ is a solution (where $x>y$), then we get another solution \[((b+2)y -x, y)  = \left( \frac{y^2 + c}{x}, y \right) \]
Claim: Either $(x,y) = (1,1)$ or $c< x^2 - y^2$.
Proof: Suppose $(x,y)\ne (1,1)$, so $xy\ge 2$. We have \begin{align*}
x^2  - y^2 = (x-y)(x+y) > (x-y)^2  \\
= bxy - c > bxy - b \\
\ge  b > c, \\
\end{align*}as desired. $\square$

Thus, if $xy>1$, $0< \frac{y^2 + c}{x} < x$ which means we have another solution with lower maximum value. Thus we can keep repeating such an operation until we get $(1,1)$ is a solution. However, now $2 - (b+2) = -c$, which means $b = c$, which we know isn't true. Therefore there are no solutions.
This post has been edited 1 time. Last edited by megarnie, May 8, 2023, 2:33 PM
Z K Y
The post below has been deleted. Click to close.
This post has been deleted. Click here to see post.
bryanguo
1032 posts
#42
Y by
nice
Suppose for the sake of contradiction the expression can take on square values. This is equivalent to \[a^2+\frac{4a^2+c}{b} = k^2,\]where $k$ is a positive integer, and $c$ takes on any value $0 \leq c <b.$ Rewrite this as $(b+4)a^2-bk^2=-c.$ Obviously, $k>a,$ so we use the clever substitution $a=\tfrac{x-y}{2}$ and $k=\tfrac{x+y}{2},$ where $x$ and $y$ take on positive integer values. Slogging through algebra: \[(b+4)\left(\frac{(x-y)^2}{4}\right) - b\left(\frac{(x+y)^2}{4}\right) = -c \implies x^2-(b+2)xy+y^2=-c.\]We now Vieta Jump \[(x,y) \to \left(y, \frac{y^2+c}{x}\right) = (y, (b+2)y-x).\]Suppose $(x,y) \neq (1,1).$ Then we prove $\tfrac{y^2+c}{x}<x.$ Observe that $xy \geq 2.$ From our quadratic in $x,$ we know \[(x-y)^2-bxy=-c>-b \implies (x-y)^2>bxy-b \geq b > c.\]But \[(x-y)^2<(x-y)(x+y)=x^2-y^2,\]implying $c<x^2-y^2.$ Therefore, if minimal $(x,y)$ were to exist, then our process terminates only when we reach the base pair $(x,y)=(1,1).$ Otherwise, using our transformation, we would be able to find a smaller pair. But when plugged back into our original equation, this yields $2-(b+2)=-c \implies b=c,$ which is absurd.
Z K Y
The post below has been deleted. Click to close.
This post has been deleted. Click here to see post.
asdf334
7585 posts
#43
Y by
Wow this solution is identical to #37 and #38. !!!!

(Also 2019 ISL N8)
Let $a$ and $b$ be two positive integers. Prove that the integer
\[a^2+\left\lceil \frac{4a^2}{b}\right\rceil\]is not a square.

Wow, this is really interesting. There are no actual theorems used here, just some clever observations and a bit of luck.

Lemma: The quantity
\[a^2+4\left\lceil \frac{a^2}{b}\right\rceil\]is not a perfect square.
Proof: Suppose otherwise. Take $k\ge 1$. Write
\[a^2+4\left\lceil \frac{a^2}{b}\right\rceil=(a+2k)^2\implies \left\lceil \frac{a^2}{b}\right\rceil = ak+k^2.\]Now comes the first clever observation. If $b\ge \frac{a}{k}$, then
\[\frac{a^2}{b}\le ak\implies  \left\lceil \frac{a^2}{b}\right\rceil\le ak\]which can't happen. Likewise if $b\le \frac{a-k}{k}$ then
\[\left\lceil \frac{a^2}{b}\right\rceil\ge \frac{a^2}{b}\ge \frac{a^2k}{a-k}>ak+k^2\]hence $b\in \left(\frac{a}{k}-1,\frac{a}{k}\right)$. As a result $k\nmid a$ and
\[b=\left\lfloor \frac{a}{k}\right\rfloor.\]Write $a=bk+c$ with $1\le c<k$. Then
\[\left\lceil \frac{a^2}{b}\right\rceil=ak+k^2\iff \left\lceil \frac{b^2k^2+2bkc+c^2}{b}\right\rceil=bk^2+kc+k^2\]so that
\[bk^2+2kc+\left\lceil \frac{c^2}{b}\right\rceil=bk^2+kc+k^2\implies k^2-kc-\left\lceil \frac{c^2}{b}\right\rceil=0.\]Well, here's nice observation #2: since we have a quadratic with integer coefficients with an integer $k$ as the root, it follows that the discriminant
\[c^2+4\left\lceil \frac{c^2}{b}\right\rceil\]is a square. But wait! This is analogous to the original expression with $a\to c$ (it's important that $c\ge 1$). If we prove that $c<a$, we should have a contradiction (classic infinite descent argument). But now, $c<k$ and so we need to prove $k\le a$, which is obvious from
\[a^2\ge \left\lceil \frac{a^2}{b}\right\rceil=ak+k^2.\]The lemma is proven.

It might seem strange that we just moved the $4$ outside, but it turns out the case where $4$ is inside is analogous. Write
\[a^2+\left\lceil \frac{4a^2}{b}\right\rceil=(a+k)^2=a^2+2ak+k^2\]and now
\[\left\lceil \frac{4a^2}{b}\right\rceil=2ak+k^2\]so we've just taken $a\to 2a$ and may proceed with the same strategy as used in the lemma. Done!
This post has been edited 2 times. Last edited by asdf334, Dec 6, 2023, 1:38 AM
Z K Y
The post below has been deleted. Click to close.
This post has been deleted. Click here to see post.
ihategeo_1969
235 posts
#44
Y by
My Vieta Jump is a bit different than the ones posted, so hopefully I am not wrong. (and if irs right, then yay my first N8! :P )

Assume not. Call $(a,b)$ good if it satisfies the above equation. Extend the domain of $a$ to non-zero integers. See that $(a,b)$ is good if and only if $(-a,b)$ is good.

Let $(a,b)=(c,d)$ be the minimal solution which works with $c>0$ (so in other words $c+d$ is minimal). Let \[X^2=c^2+\left\lceil\frac{4c^2}d\right\rceil \iff d(X^2-c^2)=4c^2+r \text{ for some $0 \leq r < d$}\]\[\iff 4c^2-2cdk+r-dk^2=0\]with $X=c+k$ with $k>0$ obviously. Let \[-c'=\frac{r-dk^2}{4c}=\frac{dk}2-c\]now see that $c'$ satisfies $X^2={(c')}^2+\left\lceil\frac{4{(c')}^2}d\right\rceil $ and since $c'$ is obviously rational, it must be an integer. And it is positive as well as $r<d \leq dk^2$. And so $(c',d)$ is also good with $c \in \mathbb{N}$ and hence we get \[c \leq c' = \frac{dk^2-r}{4c} \iff 4c^2 \leq dk^2-r \leq dk^2\]\[\implies \frac{4c^2}d \leq k^2 \iff \left\lceil\frac{4c^2}d\right\rceil \leq k^2\]\[\implies X^2={(c+k)}^2 \leq c^2+k^2\]which is a clear contradiction, as desired.
Z K Y
The post below has been deleted. Click to close.
This post has been deleted. Click here to see post.
Sammy27
83 posts
#45 • 1 Y
Y by Eka01
Storage
This post has been edited 1 time. Last edited by Sammy27, Jun 17, 2024, 8:30 PM
Z K Y
The post below has been deleted. Click to close.
This post has been deleted. Click here to see post.
S1muelJ
247 posts
#46
Y by
ihategeo_1969 wrote:
My Vieta Jump is a bit different than the ones posted, so hopefully I am not wrong. (and if irs right, then yay my first N8! :P )

Assume not. Call $(a,b)$ good if it satisfies the above equation. Extend the domain of $a$ to non-zero integers. See that $(a,b)$ is good if and only if $(-a,b)$ is good.

Let $(a,b)=(c,d)$ be the minimal solution which works with $c>0$ (so in other words $c+d$ is minimal). Let \[X^2=c^2+\left\lceil\frac{4c^2}d\right\rceil \iff d(X^2-c^2)=4c^2+r \text{ for some $0 \leq r < d$}\]\[\iff 4c^2-2cdk+r-dk^2=0\]with $X=c+k$ with $k>0$ obviously. Let \[-c'=\frac{r-dk^2}{4c}=\frac{dk}2-c\]now see that $c'$ satisfies $X^2={(c')}^2+\left\lceil\frac{4{(c')}^2}d\right\rceil $ and since $c'$ is obviously rational, it must be an integer. And it is positive as well as $r<d \leq dk^2$. And so $(c',d)$ is also good with $c \in \mathbb{N}$ and hence we get \[c \leq c' = \frac{dk^2-r}{4c} \iff 4c^2 \leq dk^2-r \leq dk^2\]\[\implies \frac{4c^2}d \leq k^2 \iff \left\lceil\frac{4c^2}d\right\rceil \leq k^2\]\[\implies X^2={(c+k)}^2 \leq c^2+k^2\]which is a clear contradiction, as desired.

:what?: :wacko:
Z K Y
The post below has been deleted. Click to close.
This post has been deleted. Click here to see post.
HamstPan38825
8866 posts
#47
Y by
this is like the most canonical Vieta jumping problem I've ever seen

Let $\frac{4a^2+r}b \in \mathbb Z$ with $0 \leq r < p-1$. The condition is equivalent to writing
\[a^2b+4a^2+r = k^2b.\]Using the substitution $(a, k) = \left(\frac{m-n}2, \frac{m+n}2\right)$ with $m > n$ positive integers, the equation rewrites as \[m^2+n^2-(b+2)mn+r=0.\]Consider the solution $(m_0, n_0)$ to this equation with $m_0+n_0$ minimal. As both $\left(\frac{n_0^2+r}{m_0}, n_0\right)$ is also a solution, it follows that $m_0^2 - n_0^2 \leq r$, implying $m_0 \leq \frac{r+1}2 < \frac{b+1}2$. In particular, $a \leq \frac b2$, so $\frac{4a^2}b \leq 2a$, i.e. $a^2 + \left \lceil \frac{4a^2}b \right \rceil < (a+1)^2$, contradiction.
Z K Y
The post below has been deleted. Click to close.
This post has been deleted. Click here to see post.
InterLoop
280 posts
#48
Y by
solved with NTguy
solution
This post has been edited 2 times. Last edited by InterLoop, Apr 24, 2025, 12:17 PM
Z K Y
N Quick Reply
G
H
=
a